LSAT and Law School Admissions Forum

Get expert LSAT preparation and law school admissions advice from PowerScore Test Preparation.

User avatar
 Dave Killoran
PowerScore Staff
  • PowerScore Staff
  • Posts: 5852
  • Joined: Mar 25, 2011
|
#45451
Complete Question Explanation
(The complete setup for this game can be found here: lsat/viewtopic.php?t=2450)

The correct answer choice is (C)

If J drives on both Wednesday and Saturday and on no other day, then according to the contrapositive of the last rule G cannot drive on Monday. Since F cannot drive on Monday from the third rule, the only driver available to drive on Monday is H. Hence, answer choice (C) is correct.

Get the most out of your LSAT Prep Plus subscription.

Analyze and track your performance with our Testing and Analytics Package.